Inscription / Connexion Nouveau Sujet
Niveau Maths sup
Partager :

égalité

Posté par
flopflop
14-09-08 à 14:40

Bonjour, il faut que je trouve les 3 entiers naturels vérifiant:

\frac{1}{a^2}+\frac{1}{b^2}+\frac{1}{c^2}=\frac{1}{4}

J'ai essayer de mettre tous sur le même dénominateur mais ça ne me simplifie pas la tâche j'aboutie à :

\frac{b^2c^2+a^2c^2+b^2+a^2}{a^2b^2c^2}=\frac{1}{4}

Mais je bloque pour la suite. J'ai remarqué que toutes les lettres jouent le même role mais je bloque.
Merci de votre aide

Posté par
xunil
re : égalité 14-09-08 à 14:46

bonjour,

par symétrie des roles tu peux supposer que 1\le a\le b\le c

\frac{1}{a^2}+\frac{1}{b^2}+\frac{1}{c^2}\le \frac{3}{a^2}

ie \frac{1}{4}\le \frac{3}{a^2}

12\ge a^2

ce qui donne des valeurs finies pour a

sans certitude

Posté par
flopflop
re : égalité 14-09-08 à 15:23

Merci de ta réponse xunil. Je ne comprend pas pourquoi tu dis que l'on a des valeurs finies pour a.
Je suis très bien ton raisonnement jusqu'à 12 a^2
on travail avec a, b, c naturels donc je devrai avoir 2\sqrt{3}=a comme valeur maximum pour a,b,c ? Mais comment trouver ces valeurs ? Faire tous les tests possible pour a,b,c [0,3]
Merci de votre aide

Posté par
flopflop
re : égalité 20-09-08 à 14:17

up ... J'ai cherché pendant 1 semaine sans rien ... assez facheux.. je ne suis pas très sur du 12\ge a^2

Merci de votre aide

Posté par
Camélia Correcteur
re : égalité 20-09-08 à 14:32

Bonjour

xunil a raison! Les entiers tels que a2 12 sont 1,2,3. Essaye lesquels conviennent!

Posté par
flopflop
re : égalité 20-09-08 à 14:34

justement ... aucun
La réponse est 3,3,6 mais j'arrive pas à le justifier :p

Posté par
Camélia Correcteur
re : égalité 20-09-08 à 14:46

Si a=1, on aurait (1/4)-(1/a2)=-3/4=1/b2+1/c2 impossible.

Pour a=2, c'est tout aussi impossible.

Reste a=3: Alors 1/b2+1/c2=5/36 avec b et c supérieurs à 3. Il n'y a que peu de possibilités!

Posté par
flopflop
re : égalité 20-09-08 à 15:06

Merci beaucoup ! je viens de trouver la solution et avec la bonne démonstration

Posté par
flopflop
re : égalité 20-09-08 à 19:46

Je me suis prononcé un peu trop vite :S
Camélia, si je suis ton raisonnement je devrai finir la démonstration comme il suit :

Reste a=3: Alors \frac{1}{b^2}+\frac{1}{c^2}=\frac{5}{36} avec b et c supérieurs ou égal à 3.

On a donc :
\frac{1}{b^2}=\frac{5}{36}-\frac{1}{c^2}
On reprend la ligne précédente on fait donc UN premier test pour:
c=3: Alors  \frac{1}{b^2}=\frac{5}{36}-\frac{1}{3^2}=\frac{1}{b^2}=\frac{1}{36}

Mais c'est ici que le problème commence :
après tout on aurait très bien pu mettre \frac{1}{b^2}=\frac{5}{36}-\frac{1}{4^2}=\frac{11}{144} ( impossible car numérateur différent de 1 )
Ou encore : \frac{1}{b^2}=\frac{5}{36}-\frac{1}{5^2}=\frac{89}{900} ( idem )
Et la ça devient intéressant : \frac{1}{b^2}=\frac{5}{36}-\frac{1}{6^2}=\frac{1}{9} ( valeur possible )

Je comprend bien qu'il faut qu'il y ai un 1 au numérateur mais pourquoi c² doit être égal à 3 ou 6 et pas à une valeur supérieur? comment justifier qu'il n'y a pas d'autres solutions en dehors de ceci? ( si tenté qu'il n'y en ai vraiment pas ...)

J'espère avoir été assez clair sur mon problème ...

Posté par
Camélia Correcteur
re : égalité 21-09-08 à 14:57

Puisque b c, on a 1/c2 1/b2, donc 2/b2 5/36 et b2 72/5 ce qui impose b=3.



Vous devez être membre accéder à ce service...

Pas encore inscrit ?

1 compte par personne, multi-compte interdit !

Ou identifiez-vous :


Rester sur la page

Inscription gratuite

Fiches en rapport

parmi 1675 fiches de maths

Désolé, votre version d'Internet Explorer est plus que périmée ! Merci de le mettre à jour ou de télécharger Firefox ou Google Chrome pour utiliser le site. Votre ordinateur vous remerciera !